Difference between revisions of "1961 AHSME Problems/Problem 28"

(SOLUTION 2)
(SOLUTION 2)
Line 27: Line 27:
 
We can clearly observe that,
 
We can clearly observe that,
  
  <math>U</math>(<math>7^1</math>)= <math>7</math>
+
  <math> </math>U<math>(</math>7^1<math>)= </math>7<math>
 
   .      .
 
   .      .
 
   .      .
 
   .      .
 
   .      .
 
   .      .
   <math>U</math>(<math>7^4)= </math>1<math>\
+
   </math>U<math>(</math>7^4)= <math>1</math> <math>  
  
and we can see by Fermat's Theorem that this cycle repeats with the cyclicity of </math>4<math>. Now </math>753<math> = </math>4k<math> + </math>1<math> </math>=><math> </math>U(<math>7</math>^{753}<math>)</math> = <math>7</math>.
+
and we can see by Fermat's Theorem that this cycle repeats with the cyclicity of </math>4<math> .                                                    
 +
 
 +
Now </math>753<math> = </math>4k<math> + </math>1<math> </math>=><math> </math>U(<math>7</math>^{753}<math>)</math> = <math>7</math>.
 
   
 
   
  

Revision as of 09:05, 31 December 2023

Problem 28

If $2137^{753}$ is multiplied out, the units' digit in the final product is:

$\textbf{(A)}\ 1\qquad \textbf{(B)}\ 3\qquad \textbf{(C)}\ 5\qquad \textbf{(D)}\ 7\qquad \textbf{(E)}\ 9$

Solution

$7^1$ has a unit digit of $7$. $7^2$ has a unit digit of $9$. $7^3$ has a unit digit of $3$. $7^4$ has a unit digit of $1$. $7^5$ has a unit digit of $7$.

Notice that the unit digit eventually cycles to itself when the exponent is increased by $4$. It also does not matter what the other digits are in the base because the units digit is found by multiplying by only the units digit. Since $753$ leaves a remainder of $1$ after being divided by $4$, the units digit of $2137^{753}$ is $7$, which is answer choice $\boxed{\textbf{(D)}}$.

SOLUTION 2

  • $Lemma$ ($Fermat's$ $Theorem$): If $p$ is a prime and $a$ is an integer prime to $p$ then we have $a^{p-1} \equiv 1\ (\textrm{mod}\ p)$.


  • Let's define $U$($x$) as units digit funtion of $x$.

We can clearly observe that,

 $$ (Error compiling LaTeX. Unknown error_msg)U$($7^1$)=$7$.      .    .      .    .      .$U$($7^4)= $1$ $and we can see by Fermat's Theorem that this cycle repeats with the cyclicity of$4$.                                                     

Now$ (Error compiling LaTeX. Unknown error_msg)753$=$4k$+$1$$ (Error compiling LaTeX. Unknown error_msg)=>$$ (Error compiling LaTeX. Unknown error_msg)U($7$^{753}$)$ = $7$.


$~GEOMETRY-WIZARD$

See Also

1961 AHSC (ProblemsAnswer KeyResources)
Preceded by
Problem 27
Followed by
Problem 29
1 2 3 4 5 6 7 8 9 10 11 12 13 14 15 16 17 18 19 20 21 22 23 24 25 26 27 28 29 30 31 32 33 34 35 36 37 38 39 40
All AHSME Problems and Solutions

The problems on this page are copyrighted by the Mathematical Association of America's American Mathematics Competitions. AMC logo.png